¿Se pueden derivar las ecuaciones de Maxwell de la ley de Coulomb y la relatividad especial?

Como ejercicio, me senté y deduje el campo magnético producido por cargas en movimiento para algunas situaciones artificiales. Empecé con la Ley de Coulomb y la Relatividad Especial. Por ejemplo, derivé el campo magnético producido por una corriente yo en un hilo infinito. Es un efecto relativista; en el marco de una carga de prueba, la densidad de electrones aumenta o disminuye en relación con la densidad de protones en el cable debido a la contracción relativista de la longitud, según el movimiento de la carga de prueba. El efecto neto es un campo de Coulomb dependiente del marco cuyo efecto sobre una carga de prueba es exactamente equivalente al de un campo magnético según la Ley de Biot-Savart.

Mi pregunta es: ¿Se pueden derivar las ecuaciones de Maxwell usando solo la Ley de Coulomb y la Relatividad especial?

Si es así, y el B -field es en todos los casos un efecto puramente relativista, entonces las ecuaciones de Maxwell se pueden reescribir sin hacer referencia a un B -campo. ¿Esto todavía deja espacio para los monopolos magnéticos?

Tengo un vago recuerdo de que, cuando estaba en la escuela secundaria, encontré un libro que hizo una licenciatura en E&M al asumir que SR es correcto desde el principio y hacer algo como esto. Sin embargo, no recuerdo el título (o si era bueno), pero si quieres ver esto resuelto en detalle, ¿podrías intentar buscarlo?
@Jeremy: el libro en el que estás pensando es probablemente Electricity and Magnetism de E. Purcell (parte de la serie Berkeley Physics). Un muy buen libro, por cierto.
¡Sí! Creo que es en el que estaba pensando.
Sí, usé este libro en la universidad. Muy buen libro. Pero no fue "hasta el final" y derivó las ecuaciones de Maxwell (si no recuerdo mal).
También necesita la suposición de que la carga es un repelente escalar y de las mismas cargas. Entonces la derivación está contenida en el libro EM de Purcell.
Sí, es posible y Haskell lo lleva a cabo con gran detalle en este documento: richardhaskell.com/files/… Véase también la siguiente discusión muy esclarecedora sobre los problemas al aplicar las ecuaciones de Maxwell a cargas fuente aceleradas.

Respuestas (13)

Las ecuaciones de Maxwell se derivan de las leyes de la electricidad combinadas con los principios de la relatividad especial. Pero este hecho no implica que el campo magnético en un punto dado sea menos real que el campo eléctrico. Muy por el contrario, la relatividad implica que estos dos campos tienen que ser igualmente reales.

Cuando se imponen los principios de la relatividad especial, el campo eléctrico mi tiene que incorporarse a un objeto que se transforma de una manera bien definida bajo las transformaciones de Lorentz, es decir, cuando cambia la velocidad del observador. Porque no existe la "fuerza eléctrica escalar", y por otras razones técnicas que no quiero explicar, mi no puede ser parte de un vector de 4 en el espacio-tiempo, V m .

En su lugar, deben ser los componentes F 0 i de un tensor antisimétrico con dos índices,

F m v = F v m
Dichos objetos, generalmente conocidos como tensores, saben cómo comportarse bajo las transformaciones de Lorentz, cuando el espacio y el tiempo rotan entre sí como lo hace obligatorio la relatividad.

los índices m , v tomar valores 0 , 1 , 2 , 3 es decir t , X , y , z . Debido a la antisimetría anterior, hay 6 componentes no equivalentes del tensor: los valores de m v puede ser

01 , 02 , 03 ; 23 , 31 , 12
Las tres primeras combinaciones corresponden a las tres componentes del campo eléctrico. mi mientras que las últimas tres combinaciones llevan la información sobre el campo magnético B .

Cuando tenía 10 años, también pensé que el campo magnético podría haber sido solo un artefacto del campo eléctrico, pero no puede ser así. En cambio, los campos eléctrico y magnético en cada punto son completamente independientes entre sí. Sin embargo, la simetría de Lorentz puede transformarlos entre sí y ambos son necesarios para que su amigo pueda transformarse en algo en un sistema inercial diferente, para que la simetría bajo el cambio del sistema inercial no se pierda.

Si solo empiezas con el mi z campo eléctrico, la componente F 03 es distinto de cero. Sin embargo, cuando impulsa el sistema en el X -dirección, mezclas la coordenada de tiempo 0 con el espacial X -coordinar 1 . En consecuencia, una parte de la F 03 el campo se transforma en el componente F 13 que se interpreta como el campo magnético B y , hasta un cartel.

Alternativamente, uno puede describir la electricidad por el potencial eléctrico ϕ . Sin embargo, la densidad de energía de la densidad de carga ρ = j 0 tiene que ser un tensor con dos índices temporales, T 00 , asi que ϕ en sí mismo también debe llevar un índice similar al tiempo. debe ser eso ϕ = A 0 para algunos 4 vectores A . Todo este 4-vector debe existir por relatividad, incluidos los componentes espaciales. A , y un nuevo campo B puede calcularse como el rotacional de A tiempo mi = ϕ A / t .

Aparentemente quería probar la ausencia de los monopolos magnéticos demostrando la ausencia del campo magnético mismo. Bueno, disculpas por haber interrumpido tu plan de investigación: no puede funcionar. Los imanes son malditamente reales. Y si está interesado, la existencia de monopolos magnéticos es inevitable en cualquier teoría consistente de la gravedad cuántica. En particular, dos polos de un imán en forma de mancuerna pueden colapsar en un par de agujeros negros que inevitablemente poseerán las cargas monopolares magnéticas (opuestas). Los agujeros negros más ligeros posibles (masa de Planck) con cargas de monopolo magnético serán "pruebas de concepto" partículas elementales pesadas con cargas magnéticas; sin embargo, a veces también pueden existir partículas más ligeras con las mismas cargas.

Entonces, ¿es cierto que las ecuaciones de Maxwell se pueden reescribir sin hacer referencia a un campo B?
No. Siempre debe haber un B -campo. Puede calcularse a partir de otro campo, como el vector potencial. A , dónde B = rizo  A , pero B debe existir y existe. Sus componentes deben ser y son independientes de las componentes del campo eléctrico. Y la relatividad implica que B existe , en lugar de que no existe. ¿Mi respuesta anterior no estaba clara o hay una razón más profunda por la que está haciendo la misma pregunta nuevamente?
Su respuesta anterior solo no estaba clara en el sentido de que no aborda por qué puedo (en algunos ejemplos artificiales, ciertamente) resolver las ecuaciones de movimiento sin referencia a un campo B. Todo lo que necesito hacer es mostrar cómo se transforma el campo E bajo los impulsos de Lorentz, y puedo hacerlo sin introducir un campo B. ¿No hice esos ejemplos correctamente, o son excepciones afortunadas porque son artificiales?
Estimado usuario1247, sí, efectivamente, la relatividad nos obliga a descubrir cómo mi se transforma bajo los impulsos de Lorentz. Y si haces bien los cálculos, y yo lo hice bien, descubrirás que mi no puede transformarse en sí mismo. Debe transformarse en otro campo, B . Incluso si empiezas con mi distinto de cero y B = 0 , el impulso de Lorentz por velocidad v creará un nuevo campo B = v × mi , y no hay manera de evitarlo. ¿Podría leer mi respuesta antes de repetir su concepto erróneo nuevamente? Gracias.
Leí tu respuesta detenidamente y todavía no respondes mis preguntas. Cuando hago mis ejemplos, en ninguna parte debo postular un "nuevo campo". Simplemente empiezo con la ley de Coulomb y SR, y hago las matemáticas, y las matemáticas muestran que una partícula experimenta fuerzas que pueden ser efectivamente descritas por un "nuevo campo". Esto es análogo a la fuerza de Coriolis. ¿La gravedad más un marco de referencia giratorio implican un nuevo "campo de Coriolis"? Por supuesto que no, pero puede ser efectivamente descrito por uno.
Realmente respeto tu conocimiento de física, Lubos, y realmente te agradezco por tomarte el tiempo de responder esta pregunta. Sé que puede responder la pregunta, pero creo que no se está tomando el tiempo para tratar de leer y comprender mi pregunta con la suficiente atención como para hacerlo.
Usted escribe: "en ninguna parte debo postular un 'nuevo campo'". - Precisamente por eso te digo que no has logrado leer mi texto porque mi texto muestra que hay que postular un nuevo campo para que la relatividad se sostenga. Ese es el punto mismo de esto, y eso es lo mismo que preguntaste. Simplemente no te gusta la respuesta, ¿verdad? El campo magnético no puede ser "efectivo" si el campo eléctrico es real. Debe ser exactamente igual de real porque están relacionados por una simetría: son partes de un tensor.
La fuerza de Coriolis no se deriva de la electricidad o el magnetismo. ¿Por qué estás mezclando estas cosas aquí? En relatividad general, sin embargo, se requiere que la gravedad sea indistinguible de la aceleración. Para hacerlo, se prueba que tiene que haber un nuevo campo que recuerde la aceleración local. Podría llamarse el campo de Coriolis, pero generalmente se le llama de manera más general, el tensor métrico. Sí, el tensor métrico también se sigue de simetrías y principios. Entonces también te equivocas sobre la fuerza de Coriolis. Esos campos pueden ser manifestaciones de cadenas, pero aun así deben ser reales e independientes.
La respuesta de Lubos es muy buena y muy precisa. Me suscribo completamente. Solo estoy un poco desconcertado por el último párrafo de la respuesta, donde dice que la existencia de monopolos magnéticos es inevitable en cualquier teoría consistente de la gravedad cuántica. El argumento dado es que dos polos de un imán en forma de mancuerna pueden colapsar en un par de agujeros negros que "inevitablemente poseerán las cargas monopolares magnéticas (opuestas)". Si rompo un imán, no obtengo dos monopolos, por supuesto. Tengo dos imanes. ¿Qué impediría que los dos agujeros negros hicieran exactamente lo mismo y se comportaran como dos imanes, sin
... cualquier monopolo alrededor? (ese es el resto del comentario del Dr. Rovelli, que fue cortado por el sistema)
Estimado @Carlo Rovelli, gracias por sus comentarios. Y sí, por supuesto, un imán (dipolo) se rompe en dos imanes (dipolos). Pero eso se debe a que los dipolos magnéticos son transportados por los espines de los electrones y solo se puede romper el imán (dipolo) a lo largo de las superficies alejadas de los electrones, por lo que siempre se obtienen dos imanes (dipolos) y ningún monopolo. Sin embargo, los agujeros negros no están "hechos de electrones", por lo que la desaparición de sus cargas monopolares magnéticas no tiene por qué ser válida, lo que significa, según el principio de Gell-Mann, que generalmente no es así.
Permítanme agregar algunas referencias. Mire, por ejemplo , arxiv.org/abs/hep-th/9404076 que se refiere al artículo de Affleck-Manton, quien generalizó el efecto Schwinger a los campos magnéticos. Al igual que el campo eléctrico de Schwinger producirá pares electrón-positrón, el campo magnético producirá pares monopolo-antimonopolo. Las condiciones que permiten esta producción son inevitables en QG, donde los monopolos pueden estar representados por agujeros negros con carga monopolar.
Me doy cuenta de que hasta ahora mis comentarios podrían haber sonado circulares. Pero la producción de pares de agujeros negros puede mostrarse mediante un instante magnético gravitacional que conduce a los dos agujeros negros con carga monopolar magnética que se alejan uno del otro. Tal instantón satisface todas las propiedades locales y de cuantización que uno pueda imponer en GR, por lo que es físico e implica que la probabilidad del proceso es distinta de cero. Cualquier regla adicional que prohíba tales instantones violaría la localidad. El instanton continúa analíticamente una solución de Ernst de 1976. También se recomienda el artículo de Garfinkle Strominger 1991.
Lubos, ¿está al tanto de esta referencia a los monopolos magnéticos en un crystal sciencedaily.com/releases/2009/09/090903163725.htm ? Una pregunta que tengo por no haber visto todavía los monopolos del big bang: ¿podría estar relacionada su rareza con el mismo mecanismo que explicaría el hecho de que macroscópicamente tampoco tenemos igualdad entre partículas y antipartículas?
@LubošMotl Hace su declaración "Y si está interesado, la existencia de monopolos magnéticos es inevitable en cualquier teoría consistente de la gravedad cuántica". ¿Implica que los físicos que estudian la gravedad cuántica creen que para que la gravedad cuántica exista también deben existir monopolos magnéticos? No había oído eso como condición para/consecuencia de la existencia de la gravedad cuántica.
Estimado @Stan, estoy seguro de que la mayoría de los expertos con más de 5000 puntos en artículos sobre gravedad cuántica estarán de acuerdo conmigo en que los monopolos magnéticos son inevitables en la gravedad cuántica. En realidad, se debe a que puede crear agujeros negros con un flujo magnético confinado: imagine que dos puntos finales de una gran barra magnética colapsan por separado en agujeros negros. Entonces, estos objetos existen en forma de agujeros negros e incluso si el agujero negro evapora todas las partículas con q metro = 0 , algo con q metro 0 tiene que ser dejado, de todos modos. Las partículas elementales monopolares magnéticas pueden identificarse con los microestados BH más ligeros
Entonces, ¿por qué no hay una contraparte de imán de gravedad en GR o monopolo de imán de gravedad?
Las cargas en el electromagnetismo y la gravedad tienen diferentes espines: representaciones bajo el grupo de Lorentz. Son las cantidades conservadas, y resultan ser la Q eléctrica como generador escalar de U(1) en electricidad, Q_m adicional como generador de S-dual U(1) en magnetismo. En gravedad, las cargas conservadas son el vector energía-momento de índice 1. El electromagnetismo y la gravedad simplemente no son exactamente iguales o isomorfos, simplemente son análogos en la mayoría de los aspectos.
El argumento sobre el par de monopolos por el colapso de un imán es erróneo. El imán está hecho de materia normal y electrones. En algún punto del colapso, el imán se fragmentará y formará dos (o más) imanes más pequeños. En cada punto del colapso, se cumplirán las ecuaciones clásicas de Maxwell (con la gravedad) y no habrá monopolos. La creación de monopolos en este proceso implica una nueva física. Entonces, el colapso gravitacional de un imán no sirve para nada para justificar los monopolos.
Te equivocas. De hecho, los monopolos son una nueva física y la creación de monopolos demuestra una nueva física. Pero el argumento es válido, por lo que es una prueba de que la física más allá de SM tiene que existir. Su afirmación de que la materia siempre se reorganiza para contener solo electrones violaría la localidad. Tu problema es que crees injustificadamente que cosas como los monopolos magnéticos no existen. Pero los agujeros negros pueden transportar cargas magnéticas, de todos modos no están descritos por el Modelo Estándar, y un colapso genérico de los imanes producirá agujeros negros cargados magnéticamente solo por GR puro, que es una aproximación correcta.
Pero parece que uno puede definir el campo magnético en términos del campo eléctrico como B i = tu / C 2 ϵ i j k α j mi k dónde tu i = tu α i es la velocidad relativa al marco en el que las cargas de la fuente no se mueven. Por lo tanto, no parece cierto que deba postularse un campo B adicional, se puede definir en términos del campo E del marco de reposo y las velocidades relativas.
¿Conocías los campos eléctricos y magnéticos cuando tenías 10 años?
Sí, @ApoorvPotnis, pero esto ni siquiera fue algo excepcional. Los campos eléctricos y magnéticos, su existencia y fundamentos, realmente se enseñan en la escuela en mi país a esa edad. Hay muchas cosas que supe muchos años antes que mis compañeros de clase, etc. pero esto no es un ejemplo. Si lo desea, puedo encontrar libros de texto para niños de 10 años que traten estos campos, etc.
@LubošMotl Usted escribe: "En cambio, deben ser los componentes F0i de un tensor antisimétrico con dos índices". ¿Puede explicar cómo sigue esto? Empecé asumiendo que las componentes del campo eléctrico tienen que ser componentes de un Tensor. Sé cómo se comportan estos componentes bajo rotaciones, así es como concluyo que los componentes del campo eléctrico tienen que ser componentes similares al espacio. Pero aquí estoy atascado: ¿Qué suposiciones necesito para mostrar que el tensor es de segundo rango y antisimétrico? ¿Empleo la propiedad de cómo se transforman las fuerzas?
Creo que LubošMotl y @user1247 han estado discutiendo sobre si el campo magnético es real. Pero la verdadera pregunta del usuario 1247 es si el monopolo magnético tiene un lugar en la relatividad y la ley de Coulomb derivó las ecuaciones de Maxwell. Luboš Motl no respondió eso en la primera parte de la respuesta. Respondió eso en la parte posterior con nueva física, que está más allá de la pregunta original del usuario 1247.
Por si a alguien le interesa, soy bastante curiosa. Y no creo que esté subespecificado. Creo que hay una respuesta correcta: physics.stackexchange.com/q/656304/307354

La respuesta de Lubos Motl es muy buena, pero creo que vale la pena decir una o dos cosas más.

Puede considerar el magnetismo simplemente como un subproducto de la electricidad, en el siguiente sentido: si asume que la Ley de Coulomb es correcta, que la relatividad especial es correcta y que la carga es un escalar de Lorentz (de modo que la carga y la densidad de corriente forman un 4- vector), entonces puede derivar todas las ecuaciones de Maxwell. (En realidad, probablemente también deba asumir que la teoría también es lineal, ahora que lo pienso). El libro de texto de nivel universitario de Purcell resuelve esto muy explícitamente de una manera agradable y agradable, y también está en libros de texto más avanzados. .

Algunos libros pasan por alto la necesidad de postular que la carga es un escalar. Al menos un libro de texto, no recuerdo cuál, lo enfatiza y presenta un caso convincente de que vale la pena prestarle atención. Una forma de ver que no es una condición trivial para imponer es considerar la analogía con la gravedad, es decir, sustituir la masa por la carga y la gravedad por el campo eléctrico, y tratar de ejecutar el mismo argumento. (Suponga campos débiles para que todo pueda tratarse como lineal si lo desea). Hay efectos "gravitomagnéticos", pero no están relacionados con la gravedad normal de la misma manera que el campo magnético está relacionado con el campo eléctrico, es decir , los análogos gravitatorios de las ecuaciones de Maxwell se ven diferentes de las ecuaciones regulares de Maxwell). Una de las razones son las diferencias de signo, por supuesto, cargas similares se repelen en un caso y se atraen en el otro. Pero una razón mayor es que la fuente de gravedad no es un escalar: su densidad no forma parte de un cuadrivector, sino de un tensor de rango 2.

Pero en un nivel más filosófico (o quizás semántico), no saltaría de este hecho a la conclusión de que el magnetismo es "simplemente" un subproducto de la electricidad. ¡Por lo menos, ese lenguaje no parece ser útil para comprender la teoría o usarla! Por ejemplo, entender cómo una onda electromagnética puede propagarse desde una galaxia lejana hasta tu ojo es mucho más fácil y natural si lo miras desde el punto de vista "habitual".

Gracias. Entonces, si, como usted dice, puede derivar todas las ecuaciones de Maxwell como un subproducto de la electricidad, parece deducirse trivialmente que uno puede escribir las ecuaciones de Maxwell sin hacer referencia a un campo B. (Así como puedo escribir, como en el ejercicio que describí, las fuerzas debidas a una corriente I sin referencia a un campo B). Esta es mi pregunta que Lubos parece negarse a querer abordar. Entiendo que esto no cambia la física, y puede que no sea la forma más parsimoniosa de expresar el electromagnetismo; solo me interesa si se puede y se ha hecho.
Sí, se puede hacer. Con suficiente esfuerzo, puede ir más allá y expresar toda la electricidad y el magnetismo sin hacer referencia a un campo E o B, tal como una ley de fuerza muy extraña y complicada entre cargas, en la que la fuerza de cada carga depende de las propiedades de la otra carga en el tiempo retardado. El libro de texto de Griffiths escribe la ley de la fuerza explícitamente en uno de los últimos capítulos. Renuncias a mucho al hacer esto: lo más importante que me viene a la mente es que no tengo idea de cómo tratarías de hablar sobre la conservación de la energía y el impulso en este idioma.
@ user1247 Tenga en cuenta que la parsimonia es extremadamente importante. Hay una infinidad de teorías que explican igual de bien el mismo fenómeno y hacen las mismas predicciones. Estamos tratando de reducir todo a lo fundamental. Podrías reemplazar todos los campos cuánticos con solo un campo bastante complicado que explicara todo exactamente igual de bien, pero a pesar de reemplazar múltiples campos con solo uno, sería mucho más complejo. Y esperamos que las leyes fundamentales que gobiernan el universo sean lo más simples posible (de hecho, "complejidad fundamental" suena como un oxímoron :D).
¿Pero la respuesta de Lubos parecía decir que también se puede derivar electricidad del magnetismo?

No es una respuesta directa a su pregunta, pero sigue siendo una sorprendente derivación de las ecuaciones de Maxwell:

La prueba de Feynman de las ecuaciones de Maxwell (FJ Dyson - Phys. Rev. A, 1989) muestra que es posible derivar las ecuaciones de Maxwell a partir de la segunda ley de movimiento y relaciones de conmutación de Newton (bajo límites no relativistas).

¡Enfriar! Una copia está disponible aquí: scribd.com/doc/168392117/… . Puede ser ilegal, dependiendo de las leyes sobre derechos de autor y uso legítimo de un país determinado.
¿Has leído los comentarios que siguen? No todas las ecuaciones se pueden derivar de ellos. Aunque es ingenioso, no está completo.
En esta prueba obtuvo la ley de Gauss para el magnetismo y la ley de Faraday , que tienen invariancia de Galileo .

Sí, puedes hacerlo, pero también necesitas usar un principio de superposición.

  1. Usted determina que la ley de Couloms,
    F = q q r | r | 3 ,
    es un caso límite de la fuerza relativista, que actúa sobre la carga q por el campo de una carga Q.
  2. Usando la transformación de Lorentz para la fuerza y ​​para el radio-vector,
    F = F + γ tu ( F v ) C 2 + Γ tu ( tu F ) C 2 ,
    r = r + Γ tu ( tu r ) C 2 γ tu t = r + Γ tu ( tu r ) C 2 ( t = 0 ) ,
    donde u es la velocidad del sistema inercial, v es la velocidad de carga, puede suponer que, en relación con el otro sistema inercial con velocidad relativa u, la fuerza se ve como
    F = q mi + q C [ v × B ] ,
    dónde
    mi = γ q r ( r 2 + γ 2 C 2 ( r tu ) 2 ) 3 2 , B = 1 C [ tu × mi ] .
    Por supuesto, el campo magnético es un efecto cinemático relativista, pero un procedimiento descrito anteriormente es la transformación cinemática relativista de la ley de Coulomb. Entonces, algunas personas cometieron un error al dar una respuesta negativa.
  3. Después de eso, utilizando los teoremas primarios del análisis vectorial y el procedimiento de regularización, puede "tomar" rot y div de las expresiones E y B anteriores. Después de eso, puedes obtener las ecuaciones de Maxwell. Debe usar el principio de superposición, cuando pasa de un campo de una carga a una distribución continua de múltiples cargas.
+1 para la respuesta tuya y de WIMP: Lubos y todos los demás, por supuesto, tienen bastante razón, pero, por supuesto, depende de lo que quieras decir con derivar. Con los postulados del OP tratados como un sistema de axiomas formal: por supuesto que no puedes; desde el punto de vista de un físico, donde asumes otras cosas "razonables" como la linealidad, por supuesto que puedes.
Tenga en cuenta que esto no es realmente una prueba de las ecuaciones de Maxwell. La relatividad especial solo se aplica a cargas que tienen (y siempre han tenido) velocidad constante, sin embargo, las ecuaciones de Maxwell pueden describir más que eso. Por ejemplo, en 3. demuestras que aplicar div a E conduce a una de las ecuaciones de Maxwell. Sin embargo, no está claro si esto todavía funciona si la(s) carga(s) se aceleraron en cualquier momento y los campos retardados entran en juego. Entonces, esta es una suposición adicional de su prueba, que aún se mantiene para cargas aceleradas.

Sé que Purcell y otros han usado la simetría de Lorentz como recurso pedagógico para motivar la introducción de campos magnéticos, pero no recuerdo haber visto nunca una derivación axiomática de las ecuaciones de Maxwell. Podría ser un ejercicio interesante ver con precisión qué suposiciones más allá de la simetría de Lorentz y la Ley de Coulomb son necesarias para reconstruir las ecuaciones de Maxwell.

Los campos B no son campos ficticios

Si conoce los campos eléctricos y magnéticos en un marco inercial, puede determinar los campos eléctricos y magnéticos en cualquier otro marco a través de la transformación de Lorentz. Si el campo magnético se desvanece en un marco inercial dado, podría pensar que los efectos magnéticos en otros marcos son ficticios. Sin embargo, no siempre es posible encontrar un marco en el que desaparezcan los campos magnéticos. La forma más rápida de ver esto es notar que E ^ 2 - B ^ 2 c ^ 2 es una cantidad invariante de Lorentz ( ver Wikipedia). Si encontramos que B ^ 2 > E ^ 2/c ^ 2 en un punto del espacio-tiempo dado en un marco inercial dado, se deduce que B ^ 2 > 0 en ese punto en todos los marcos inerciales. De hecho, podría comenzar en un marco donde el campo eléctrico desaparece pero el campo magnético no; los campos eléctricos observados en otros marcos podrían entonces considerarse ficticios.

En general, ni el campo eléctrico ni el campo magnético pueden desaparecer bajo un impulso de Lorentz. Para ver esto rápidamente, tenga en cuenta que el producto punto del vector de campo E con el vector de campo B en un punto del espacio-tiempo dado es una cantidad invariante de Lorentz ( ver Wikipedia ). Si este producto escalar es distinto de cero en un punto del espacio-tiempo dado en un marco de inercia dado, los vectores de campo eléctrico y magnético serán distintos de cero en ese punto del espacio-tiempo en todos los marcos de inercia.

Como señaló Einstein, puede comprender el movimiento de una partícula cargada al referirse al campo eléctrico en el marco de reposo de esa partícula. Sin embargo, si tiene varias partículas con diferentes velocidades, debe realizar un seguimiento del campo eléctrico en el marco de reposo instantáneo de cada partícula. Dado que los impulsos de Lorentz mezclan el campo E con el campo B, la única forma de realizar un seguimiento del campo E en el marco de reposo de cada una de sus partículas en términos de cantidades locales en un marco inercial es por referencia al campo E y el B campo.

Localidad

Incluso si es posible, no me queda claro que sería deseable utilizar la ley de Coulomb como un axioma en la teoría electromagnética. Las ecuaciones de Maxwell explican el movimiento de las partículas haciendo referencia a los grados de libertad locales, los campos. La ley de Coulomb, por otro lado, es una forma de acción a distancia y es manifiestamente no local.

Ciertamente, es posible reescribir los campos E y B en términos de integrales sobre la densidad de carga y la densidad de corriente (no puedo publicar otro enlace, así que busque en Google "ecuaciones de Jefimenko"), y luego use estas expresiones para interpretar las fuerzas electromagnéticas como una forma de acción retardada a distancia. Sin embargo, para obtener estas expresiones se requieren supuestos sobre las condiciones de contorno en los campos E y B. Siempre podemos obtener otra solución válida de las ecuaciones de Maxwell simplemente cambiando las condiciones de contorno en los campos, lo que demuestra que los campos tienen una existencia independiente y no son meras variables contables para simplificar una interacción no local más fundamental.

Monopolos

Como suele escribirse, las ecuaciones de Maxwell no contienen términos correspondientes a la carga magnética, pero sería consistente agregar dichos términos. De hecho, Dirac demostró que la cuantificación de la carga eléctrica podría deberse a la existencia de monopolos magnéticos (no puedo publicar otro enlace, así que busque en Google "condición de cuantificación de dirac de monopolo magnético"). Las ecuaciones de Maxwell no nos dicen si existen o podrían existir monopolos magnéticos, pero la cuantización de la carga eléctrica podría ser evidencia de que existen monopolos magnéticos en algún lugar del universo.

En mi respuesta, vinculé un documento de Hans de Vries donde hizo lo que dice el usuario 1247, y puede verificar su validez. En motionmoutain ch 18 - Motion in GR, encontramos también que GravitoElectric es un campo fundamental y GravitoMagnetic es un efecto relativista por la misma razón. El movimiento lo induce. Para mi es una fuerza y ​​no un 'campo' (no hay campo de Coriolis, sino fuerza). ¿Cómo la partícula puede hacer esto? "múltiples partículas con diferentes velocidades, necesita realizar un seguimiento del campo eléctrico en el marco de reposo instantáneo de cada partícula" y 'por adelantado'?

No puedes. B no es solo un efecto secundario relativista de E. Jackson, Electrodynamics , Sección 12.2 tiene una buena discusión, en la que refuta las "pruebas" dadas en algunos textos de pregrado.

"La confusión surge principalmente porque las propiedades de transformación de Lorentz de la fuerza son tales que aparece un término de fuerza de tipo magnético cuando la fuerza en un marco de inercia se expresa en términos de la fuerza en otro marco. Es tentador dar este término de fuerza extra una existencia independiente y así identificar el campo magnético como una entidad separada. Pero tal paso no se justifica sin suposiciones adicionales".

Jackson pasa a exhibir un contraejemplo explícito, basado en un potencial escalar de Lorentz. Este campo se parece a la electrostática (¡o incluso a la gravitación newtoniana!) en el límite no relativista. También tiene "una fuerza aparente de tipo magnético. Pero no hay una entidad independiente B ". Entonces, en esta "teoría", B es solo un efecto relativista, pero esta teoría no se aplica a la naturaleza.

12.2? ¿Cinemática de los productos de descomposición de una partícula inestable? ¿O estoy leyendo Electrodinámica equivocada por Jackson equivocado?
@Ruslan, mi referencia es Jackson, Classical Electrodynamics, 2.ª ed. El encabezado de la sección 12.2 es "Sobre la cuestión de obtener el campo magnético, la fuerza magnética y las ecuaciones de Maxwell a partir de la ley de Coulomb y la relatividad especial".
El título del capítulo 12 es "Dinámica de partículas relativistas y campos electromagnéticos".
Hmm, estaba probando la primera edición. Pero esto tampoco está presente en la tercera. Y no pude encontrar el 2do. De todos modos, hay un artículo de seguimiento de Kobe, que afirma que la suposición que falta para derivar las ecuaciones de Maxwell de la ley de Coulomb es que la carga es un escalar conservado. De todos modos, todavía me gustaría leer el contraejemplo de Jackson que cita, así que si tiene algún enlace a la segunda edición descargable, estaría muy agradecido si lo publicara.
Esta respuesta es útil, pero es un resumen inexacto de lo que dice Jackson. Busqué el pasaje relevante de la segunda edición. No dice que todas esas pruebas sean incorrectas, sino que se requiere cierto cuidado al explicar algunas suposiciones ocultas. Da varias referencias a tratamientos que, según él, explican correctamente los supuestos: D. H. Frisch y L. Wilets, Am. J. física. 24, 574 (1956). JR Tessman, Am. J. física. 34, 1048 (1966). M. Schwartz, Principios de electrodinámica, McGraw-Hill, Nueva York (1972), cap. 3.
@BenCrowell, después de estudiar su comentario, sigo pensando que resumí a Jackson con precisión para los fines de esta pregunta. Ciertamente no discuto que hay algunos buenos desarrollos por ahí; en particular, conozco y amo a Schwartz, mi texto de pregrado. Sin embargo, la pregunta no pedía referencias a buenos tratamientos. Ahora puede ser que me esté perdiendo algunas implicaciones involuntarias de mi texto: siéntete libre de mejorar como mejor te parezca.

Con la ley de Coulomb y la relatividad especial puedes derivar la ley de Ampere, que te da la magnetostática. Lo que falta para la electrodinámica es la corriente de desplazamiento ( 1 C 2 mi t ), que es una fuente de campo magnético que surge del campo eléctrico variable en el tiempo, y no un resultado del movimiento de la carga eléctrica.

La relatividad tiene sólo dos postulados:

  1. Las leyes de la física son las mismas en todos los marcos de referencia inerciales.
  2. Todos los observadores inerciales miden la misma velocidad de la luz en el vacío.

La relatividad, por sí misma, no exige que los campos eléctricos (o el potencial eléctrico para el caso) deban viajar a la velocidad de la luz. Para derivar las ecuaciones de Maxwell, necesita un postulado adicional, y lo proporciona la ecuación de onda (para el potencial eléctrico) en la Sección 4 de la referencia en la respuesta de Helder. Sin este postulado adicional (que los cambios en el potencial eléctrico se propagan a la velocidad de la luz), no se puede derivar la corriente de desplazamiento solo de la ley de Coulomb y la relatividad.

pero la luz ES campo electromagnético, y viceversa
@Helder Bueno, si asume la relatividad especial para derivar las ecuaciones de Maxwell de la manera discutida aquí, obviamente no puede referirse a la velocidad de la luz inicialmente porque no se define qué es la luz hasta que haya terminado con su derivación. Sin embargo, lo que puede hacer, y de hecho debe hacer, es suponer que hay una velocidad v que parece ser el mismo en todos los marcos de referencia. (Esto es básicamente lo que distingue a la mecánica galileana de SR). Es decir, formula SR usando esta velocidad v (en lugar de la velocidad de la luz C ) y derivar las ecuaciones de Maxwell como se indica en...
...las otras publicaciones aquí. Pero en algún momento tendrá que hacer la suposición de que v es precisamente la velocidad a la que se propagan las perturbaciones del campo electromagnético. Entonces JxB tiene toda la razón.

de Hans de Vries (*):

La derivación más simple y completa del magnetismo como un efecto secundario relativista de la electroestática.

Utiliza únicamente el campo Electrostático y la no simultaneidad para obtener el Campo Magnético. Lo explica mejor que Purcell.

El campo magnético es un efecto secundario del movimiento en el campo eléctrico.

(*) Hans de Vries tiene un libro en línea muy interesante (aún no terminado) en su sitio, y ofrece otra perla, no relacionada con esta publicación, pero me siento obligado a compartir: La contracción de Lorentz es un efecto real y no solo 'un efecto referencial' como estamos tentados a creer.

@JxB No puedo comentar su respuesta y citar "Para derivar las ecuaciones de Maxwell, necesita un postulado adicional, y lo proporciona la ecuación de onda (para el potencial eléctrico) en la Sección 4 de la referencia en la respuesta de Helder. Sin este postulado adicional (que los cambios en el potencial eléctrico se propagan a la velocidad de la luz), no se puede derivar la corriente de desplazamiento solo de la ley de Coulomb y la relatividad".
@JxB continúa con el comentario anterior (problema con la tecla Intro frente a ShiftEnter, lo siento) campo eléctrico = luz No se puede disociar el campo eléctrico de la luz. "a velocidad c" aquí en.wikipedia.org/wiki/Electric_field electr 28 veces aquí: en.wikipedia.org/wiki/Photon_polarization explore Radiation2D.exe desde aquí www-xfel.spring8.or.jp No tengo ninguna duda de que el campo eléctrico y la gravedad se propaguen a una velocidad c.
@JxB citando "que surge del campo eléctrico variable en el tiempo, y no como resultado del movimiento de la carga eléctrica". El movimiento es un concepto relativo y un campo eléctrico 'variable en el tiempo' es siempre el resultado de cargas en movimiento.
para mantener las referencias juntas: descripción del método Radiation2D.exe: "NUEVO MÉTODO MATEMÁTICO PARA EL CAMPO DE RADIACIÓN DE LA CARGA EN MOVIMIENTO" por T. Shintake accelconf.web.cern.ch/accelconf/e02/PAPERS/WEPRI038.pdf
El enlace está roto ahora

No, no puedes. Por varias razones. Primero, si tiene E, para obtener el campo B, necesita suposiciones adicionales sobre la estructura de la teoría, es decir, con más detalle, el tensor de intensidad de campo, consulte la respuesta anterior de Lubos. Pero además de esto, incluso si tuviera la solución para una carga puntual, para obtener las ecuaciones de Maxwell necesita saber más que solo tener una solución. Por ejemplo, que son lineales, de segundo orden, y cuál es el grupo de simetría. Y si ha agregado eso, puede derivar las ecuaciones de Maxwell a partir de estas suposiciones de todos modos sin siquiera comenzar con el campo de Coulomb.

Tengo que estar de acuerdo en que se necesitan algunas suposiciones adicionales.
+1 para la respuesta tuya y de PhysiXxx: Lubos y todos los demás, por supuesto, tienen razón, pero, por supuesto, depende de lo que quieras decir con derivar. Con los postulados del OP tratados como un sistema de axiomas formal: por supuesto que no puedes; desde el punto de vista de un físico, donde asumes otras cosas "razonables" como la linealidad, por supuesto que puedes.

Sí. Ver Principios de electrodinámica de Melvin Schwartz. Deriva toda la electrodinámica, incluidas las ecuaciones de Maxwell, de la ley de Coulomb y la relatividad especial.

Las respuestas que consisten solo en un enlace o recomendación para lectura externa son explícitamente malas respuestas.
Casi lo logró, pero en un punto en el que es difícil cerrar la brecha, se acercó a Dios. En la página 127, dijo: "Al apreciar esto (el tensor antisimétrico es económico), Dios naturalmente eligió el tensor antisimétrico como su medio de expresión". Tenga en cuenta que me encanta este libro. Es muy conciso. Lo estudié después de Griffith.

La respuesta de Luboš Motl es de alguna ayuda, ya que muestra cómo incorporar el tipo de ideas que ofrece la relatividad, pero, sin embargo, comienza con su conclusión general, y esa conclusión es incorrecta. Es incorrecto en gran parte por las razones brevemente indicadas en la respuesta de WIMP.

La pregunta es importante, y es importante obtener la respuesta correcta. La pregunta es:

¿Se pueden derivar las ecuaciones de Maxwell usando solo la ley de Coulomb y la relatividad especial?

La respuesta es: no, porque se pueden inventar muchas otras teorías de campos que respeten la Relatividad Especial, de modo que reproduzcan la Ley de Coulomb en el marco inercial de una carga puntual dada.

Sin embargo, lo que se puede decir es que el electromagnetismo clásico (es decir, la ecuación de Maxwell y la ecuación de fuerza de Lorentz, o cualquier formulación equivalente a esto, como una formulación de Lagrangian) se encuentra entre las teorías de campo más simples que respetan la Relatividad Especial e incluyen la ley de Coulomb. La definición de 'más simple' aquí es ciertamente imprecisa.

La razón principal por la que no puede derivar Maxwell de 'Coulomb + SR' es que no sabría si incluir efectos de aceleración en la relación entre potenciales y cargas.

Ahora voy a 'levantar la tapa' un poco sobre la física teórica aquí. Una muy buena (no la única) forma matemática de asegurarse de que cualquier parte de la física respete la Relatividad Especial (SR) es restringirse a expresiones tensoriales en todo lo que proponga y escriba. Aquí 'tensorial' incluye tensores de rango cero, es decir, escalares, pero no cualquier escalar antiguo: serían escalares invariantes de Lorentz. También incluye 4 vectores y tensores de rango segundo y superior. Al tomar derivadas, usa el operador de gradiente covariante a , y luego tiene un juego de herramientas para construir ecuaciones diferenciales que respetan SR

Entonces, la teoría de campo 'más simple' podría ser tal que las partículas pueden tener una propiedad escalar invariante de Lorentz llamada carga q , y la fuerza sobre una partícula cargada es independiente de la 4-velocidad tu a de la partícula El problema es que rápidamente encuentras que en tal teoría la fuerza sobre una partícula no puede cambiar la velocidad de una partícula sin cambiar también su masa. Explorando más, intentas permitir que las 4 fuerzas F a ser dependiente de la velocidad 4 a través de una ecuación lineal simple que involucra un campo escalar ϕ , como F a = q ϕ tu q (?). Todavía no es bueno (cambios masivos nuevamente). Así que te ves obligado a probar un tensor de segundo rango F a b para el campo, porque es la cosa más simple, aparte de un escalar, que puede tomar un cuadrivector tu a como entrada y devolver una fuerza de 4 vectores:

F a = q F a m tu m

Ahora está bien: la fuerza conserva la masa mientras F a b es antisimétrico. ¡Bueno! Un tensor antisimétrico es el tipo más simple de tensor de segundo orden. Lo siguiente que queremos es una ecuación diferencial para este campo: prueba lo más simple, que es tomar la divergencia, y estarás en el buen camino hacia las ecuaciones de Maxwell. Si ahora incorporamos la ley de Coulomb (y aquí es donde entra en juego), entonces tiene la garantía de obtener dos de las ecuaciones de Maxwell si restringe el término fuente en su ecuación diferencial a un solo término proporcional a la densidad de carga y 4-velocidad. . La ley de Coulomb en sí misma no le dice que no agregue más términos relacionados con 4-aceleración.

Con este enfoque no llegamos inexorablemente a las ecuaciones de Maxwell, pero sí encontramos que son posiblemente las más simples que incluyen la propiedad de conservación de la carga y que permiten una fuerza que conserva la masa (en lenguaje técnico, una fuerza pura ).

Entre otras teorías de campo que uno encuentra, hay una que se parece mucho a Maxwell pero incluye monopolos magnéticos. Esto surge de forma muy natural, en el tratamiento teórico, y es ciertamente una seria candidata a posibilidad de cómo funciona realmente el mundo físico. Es algo menos simple en el sentido de que se pierde la agradable propiedad de escribir el tensor de campo como un cuadriplicador de un campo de 4 vectores (el 4-potencial), y la teoría ya no respeta la simetría bajo la inversión del espacio (paridad). Vea el libro de Jackson sobre electromagnetismo para una discusión. Si de hecho hay monopolos magnéticos, como sugieren muchas versiones de la teoría cuántica de campos, entonces el enigma es por qué los monopolos eléctricos son mucho más abundantes que los monopolos magnéticos.

Sin embargo, me gustaría subrayar que este problema del monopolo magnético está lejos de ser la única razón por la que las ecuaciones de Maxwell no se pueden derivar completamente de la ley de Coulomb y la SR. Las otras razones incluyen que uno puede imaginar fácilmente que las ecuaciones de campo involucran derivadas de orden superior del movimiento. de la partícula; SR por sí solo no puede decirle que no lo hacen. Al comenzar con un enfoque lagrangiano, se pueden introducir restricciones adicionales, como la invariancia que conduce a leyes de conservación, y luego el electromagnetismo está restringido de manera bastante estricta, pero aún no del todo. Fundamentalmente, lo que SR puede decirle es que un campo que proporciona una fuerza independiente de la velocidad de un cuerpo no puede ser la historia completa de la física. Tal campo (como el campo eléctrico) debeestar asociado con otros efectos que dependen de la velocidad de un cuerpo.

Hay pocos artículos que muestran que la ecuación de conservación/continuidad de la carga eléctrica es suficiente para derivar todo el conjunto de ecuaciones de Maxwell. Consulte esta referencia y citas, por ejemplo; https://pdfs.semanticscholar.org/3251/31eadb62c8fdfdaaad7b21a308992ff3a4d2.pdf''Cómo obtener la forma covariante de las ecuaciones de Maxwell a partir de la ecuación de continuidad... Por lo tanto, un proceso circular parece inevitable en el electromagnetismo: ρ y J implican E y B que, a su vez, implican nuevos ρ1 y J1, y pronto. Debido a esta característica circular, no está claro si E y B (que satisfacen las ecuaciones de Maxwell) son consecuencia de ρ y J (que satisfacen la ecuación de continuidad) o viceversa. Según el árbitro parece cuestión de gustos decir cuál es consecuencia del otro. En otras palabras: del comentario del árbitro podríamos concluir que la conexión entre las fuentes y los campos es un poco como el problema del huevo y la gallina: ¿quién fue primero?''.

También a partir de la ley de Coulomb para la electricidad estática y teniendo en cuenta el hecho de que la acción no puede viajar más rápido que la luz, el uso de la integral retardada produce el conjunto completo de ecuaciones de Maxwell. https://en.wikipedia.org/wiki/Li%C3%A9nard%E2%80%93Wiechert_potential .

Por lo tanto, el retardo en sí da lugar a una fuerza que es normal al movimiento (velocidad), proporcional a él y que decae como el inverso del cuadrado de la distancia; ese es el campo magnético por definición. También da lugar a una fuerza de dos componentes: campos eléctricos y magnéticos proporcionales a la aceleración, que decaen solo como el inverso de la distancia (no el inverso al cuadrado) y esto es radiación por definición. Por lo tanto, se puede deducir que el magnetismo y la radiación son fenómenos emergentes causados ​​por la finitud de la velocidad de propagación de las fuerzas involucradas.

Algunas respuestas apuntaron a una conexión con Gravito-magnetismo y relatividad. Creo que esto se debe al hecho de que la ley de gravitación de Newton se puede tratar de manera similar a la ley de Coulomb, dando lugar a un conjunto de ecuaciones similares a las ecuaciones de Maxwell. Estas son las ecuaciones gravito-magnéticas y, de hecho, también se pueden derivar de la relatividad general para campos débiles. https://en.wikipedia.org/wiki/Gravitoelectromagnetismo

Según entiendo su idea, está preguntando si es posible recuperar todas las ecuaciones de Maxwell solo usando transformaciones de Lorentz y usando la existencia de campo eléctrico. La respuesta es no. Un ejemplo heurístico es este: si tiene un cable unidimensional circular con una corriente variable yo ( t ) , no existe una transformación de Lorentz para producir el campo magnético de este sistema a partir solo de un campo eléctrico, porque la carga eléctrica del alambre circular se mueve de forma no inercial.

Pienso lo contrario. Para cada uno de los segmentos infinitesimales del cable, para un breve intervalo de tiempo infinitesimal delta t, en el marco inercial en movimiento cuya velocidad es la misma que la de los electrones en el cable en ese caso, podemos hablar de la transformación de Lorentz que conduce a Maxwell ecuaciones Además, el potencial 4 solo depende de la corriente instantánea y no de sus derivados, por lo que podemos hacer eso.